Problemi

Vuoi proporre i tuoi esercizi? Qui puoi farlo!!

Moderatore: tutor

Bloccato
metafisic
Messaggi: 41
Iscritto il: 01 gen 1970, 01:00
Località: Franconia

Messaggio da metafisic »

1-Mostrare che esistono infiniti numeri della forma n!-1 che non sono primi
<BR>
<BR>2-Sia I={1, 2,..., n} e sia P(I) l\'insieme della parti di I.
<BR>
<BR>Quante sono le funzioni f<IMG SRC="images/forum/icons/icon_razz.gif">(I)--->I:
<BR>
<BR>f(AintB)=min(f(A), f(B)), per ogni A, B in P(I)
<BR>
<BR>[addsig]
La compactesse est metaphisique.
Avatar utente
HiTLeuLeR
Messaggi: 1874
Iscritto il: 01 gen 1970, 01:00
Località: Reggio di Calabria

Messaggio da HiTLeuLeR »

<!-- BBCode Quote Start --><TABLE BORDER=0 ALIGN=CENTER WIDTH=85%><TR><TD><font size=-1>Quote:</font><HR></TD></TR><TR><TD><FONT SIZE=-1><BLOCKQUOTE>
<BR>On 2005-01-04 21:59, metafisic wrote:
<BR>1-Mostrare che esistono infiniti numeri della forma n!-1 che non sono primi
<BR></BLOCKQUOTE></FONT></TD></TR><TR><TD><HR></TD></TR></TABLE><!-- BBCode Quote End -->
<BR>Per il teorema di Wilson, se p è un primo intero > 0: p | (p-1)!+1. <!-- BBCode Start --><I>Ergo</I><!-- BBCode End -->, nondimeno: p | (p-2)! - 1. Ne seguita l\'asserto, pur di considerare che i primi naturali sono in numero infinito, e che definitivamente in N: n! - 1 > n+2...<font color=white><BR><BR>[ Questo Messaggio è stato Modificato da: HiTLeuLeR il 04-01-2005 23:42 ]
metafisic
Messaggi: 41
Iscritto il: 01 gen 1970, 01:00
Località: Franconia

Messaggio da metafisic »

...definitivamente mitico[addsig]
La compactesse est metaphisique.
Avatar utente
HiTLeuLeR
Messaggi: 1874
Iscritto il: 01 gen 1970, 01:00
Località: Reggio di Calabria

Messaggio da HiTLeuLeR »

Indi, comunque mitico da un certo punto in poi, su su verso l\'infinito... <IMG SRC="images/forum/icons/icon21.gif">
Avatar utente
Marco
Site Admin
Messaggi: 1331
Iscritto il: 01 gen 1970, 01:00
Località: IMO '93

Messaggio da Marco »

Ciao.
<BR><!-- BBCode Quote Start --><TABLE BORDER=0 ALIGN=CENTER WIDTH=85%><TR><TD><font size=-1>Quote:</font><HR></TD></TR><TR><TD><FONT SIZE=-1><BLOCKQUOTE>
<BR>On 2005-01-04 21:59, metafisic wrote:
<BR>2-Sia I={1, 2,..., n} e sia P(I) l\'insieme della parti di I.
<BR>
<BR>Quante sono le funzioni f<IMG SRC="images/forum/icons/icon_razz.gif">(I)--->I:
<BR>
<BR>f(AintB)=min(f(A), f(B)), per ogni A, B in P(I)
<BR></BLOCKQUOTE></FONT></TD></TR><TR><TD><HR></TD></TR></TABLE><!-- BBCode Quote End -->
<BR>Mah... ad occhio e croce, direi la somma delle prime n potenze n-esime.
<BR>
<BR>Ciao. M.[addsig]
[i:2epswnx1]già ambasciatore ufficiale di RM in Londra[/i:2epswnx1]
- - - - -
"Well, master, we're in a fix and no mistake."
Avatar utente
HiTLeuLeR
Messaggi: 1874
Iscritto il: 01 gen 1970, 01:00
Località: Reggio di Calabria

Messaggio da HiTLeuLeR »

Ciao anche a te, marco, e tanti auguri! :-)
<BR>
<BR><!-- BBCode Quote Start --><TABLE BORDER=0 ALIGN=CENTER WIDTH=85%><TR><TD><font size=-1>Quote:</font><HR></TD></TR><TR><TD><FONT SIZE=-1><BLOCKQUOTE>
<BR>On 2005-01-05 08:24, marco wrote:
<BR>[...] ad occhio e croce, direi la somma delle prime n potenze n-esime.
<BR></BLOCKQUOTE></FONT></TD></TR><TR><TD><HR></TD></TR></TABLE><!-- BBCode Quote End -->
<BR>...ad occhio e croce direi invece che qualcosa non mi quadra... :-P
<BR>
<BR>Se n = 2: P(I) = {∅, {1}, {2}, I}. Poniamo:
<BR>
<BR><font color=white>ii</font>i) f(∅) = f({1}) = f({2}) = f(I) = 1;
<BR><font color=white>i</font>ii) f(∅) = f({1}) = f({2}) = 1; f(I) = 2;
<BR>iii) f(∅) = f({1}) = 1; f({2}) = f(I) = 2;
<BR>iv) f(∅) = f({2}) = 1; f({1}) = f(I) = 2;
<BR><font color=white>i</font>v) f(∅) = 1; f({1}) = f({2}) = f(I) = 2;
<BR>vi) f(∅) = f({1}) = f({2}) = f(I) = 2.
<BR>
<BR>Salvo che non abbia preso una grossa cantonata (...), ogni funzione dell\'elenco precedente soddisfa le richieste del problema, per cui...
<BR>
<BR>-------
<BR>
<BR>\"...sbaglio oppure 6 ≠ 1<sup>2</sup> + 2<sup>2</sup>?!?\" - HiTLeuLeR<font color=white><BR><BR>[ Questo Messaggio è stato Modificato da: HiTLeuLeR il 05-01-2005 11:45 ]
Avatar utente
FrancescoVeneziano
Site Admin
Messaggi: 606
Iscritto il: 01 gen 1970, 01:00
Località: Genova
Contatta:

Messaggio da FrancescoVeneziano »

<!-- BBCode Quote Start --><TABLE BORDER=0 ALIGN=CENTER WIDTH=85%><TR><TD><font size=-1>Quote:</font><HR></TD></TR><TR><TD><FONT SIZE=-1><BLOCKQUOTE>
<BR>On 2005-01-05 11:43, HiTLeuLeR wrote:
<BR>
<BR>Se n = 2: P(I) = {∅, {1}, {2}, I}. Poniamo:
<BR>
<BR><font color=white>ii</font>i) f(∅) = f({1}) = f({2}) = f(I) = 1;
<BR><font color=white>i</font>ii) f(∅) = f({1}) = f({2}) = 1; f(I) = 2;
<BR>iii) f(∅) = f({1}) = 1; f({2}) = f(I) = 2;
<BR>iv) f(∅) = f({2}) = 1; f({1}) = f(I) = 2;
<BR><font color=white>i</font>v) f(∅) = 1; f({1}) = f({2}) = f(I) = 2;
<BR>vi) f(∅) = f({1}) = f({2}) = f(I) = 2.
<BR>
<BR>Salvo che non abbia preso una grossa cantonata (...), ogni funzione dell\'elenco precedente soddisfa le richieste del problema, per cui...
<BR>
<BR></BLOCKQUOTE></FONT></TD></TR><TR><TD><HR></TD></TR></TABLE><!-- BBCode Quote End -->
<BR>
<BR>Il caso che hai etichettato con (v) non va bene, perché
<BR>1= f(∅) = f({1}int{2})=min{2,2}=2
<BR>
<BR>La soluzione data da marco è corretta e si basa sull\'osservazione che i valori della funzione su I e sugli n insiemi della forma I \\ {singoletto} determinano univocamente la funzione su tutte le parti di I tramite le loro intersezioni.
<BR>Quindi fissando f(I)=i ad ogni insieme I \\ {singoletto} posso assegnare un valori tra 1 ed i in modo indipendente, per un totale di i^n possibili scelte, da sommare per i che va da 1 a n.
<BR>
<BR>CaO
<BR>Francesco Veneziano
<BR>
<BR>[ Questo Messaggio è stato Modificato da: FrancescoVeneziano il 05-01-2005 12:26 ]
<BR>maledetti smilies...<BR><BR>[ Questo Messaggio è stato Modificato da: FrancescoVeneziano il 05-01-2005 12:28 ]
Wir müssen wissen. Wir werden wissen.
Avatar utente
Marco
Site Admin
Messaggi: 1331
Iscritto il: 01 gen 1970, 01:00
Località: IMO '93

Messaggio da Marco »

<!-- BBCode Quote Start --><TABLE BORDER=0 ALIGN=CENTER WIDTH=85%><TR><TD><font size=-1>Quote:</font><HR></TD></TR><TR><TD><FONT SIZE=-1><BLOCKQUOTE>
<BR>On 2005-01-05 11:43, HiTLeuLeR wrote:
<BR>Ciao anche a te, marco, e tanti auguri!
<BR>[...]
<BR>...ad occhio e croce direi invece che qualcosa non mi quadra...
<BR>[...]
<BR>v) f(∅ ) = 1; f({1}) = f({2}) = f(I) = 2;
<BR>Salvo che non abbia preso una grossa cantonata (...)
<BR></BLOCKQUOTE></FONT></TD></TR><TR><TD><HR></TD></TR></TABLE><!-- BBCode Quote End -->
<BR>Ciao, Hit. Ricambio gli auguri.
<BR>
<BR>Beh, direi che vale il caso della cantonata.
<BR>
<BR>La (v) non va bene. Infatti, se prendi A = {1} e B = {2}, allora
<BR>f( {} ) = f( A int B ) = min( f(A), f(B) ) = 2, non 1.
<BR>
<BR>Ciao. M.
<BR>
<BR>EDIT: Fregato sul tempo!! E bruciata anche la soluzione. E\' proprio come dice F.V. Unica avvertenza: verificare che tutte le fz che dice lui vadano bene. [OT]Ma voi lo vedete il simbolo di vuoto? Io vedo solo un quadrato...
<BR>
<BR>
<BR>\"Maybe. But so great a claim will need to be established, and clear proofs will be required.\"<BR><BR>[ Questo Messaggio è stato Modificato da: marco il 05-01-2005 12:43 ]
[i:2epswnx1]già ambasciatore ufficiale di RM in Londra[/i:2epswnx1]
- - - - -
"Well, master, we're in a fix and no mistake."
Avatar utente
Marco
Site Admin
Messaggi: 1331
Iscritto il: 01 gen 1970, 01:00
Località: IMO '93

Messaggio da Marco »

... e dato che non ha più senso tenermela per me, ecco a voi la soluzione:
<BR>
<BR>Lemmino:
<BR>
<BR>Dimostro per induzione su k che:
<BR>
<BR>f( int[j=1..k](A_j) ) = min[j]( f(A_j) ).
<BR>
<BR>Per k = 1 è ovvio. Per k = 2 è l\'ipotesi.
<BR>
<BR>Passo induttivo:
<BR>
<BR>f( int[j=1..k+1](A_j) ) = f( int[j=1..k](A_j) int A_k+1 ) =
<BR>min[ f( int[j=1..k](A_j) ), f(A_k+1) ] [per ipotesi] =
<BR>min[ min[j]( f(A_j) ), f(A_k+1) ] [ip. induttiva] =
<BR>min[j=1..k+1]( f(A_j) ). []
<BR>
<BR>--
<BR>
<BR>Quindi l\'ipotesi è vera per intersezioni di lunghezza qualunque.
<BR>
<BR>Sia ora X = {1,2,...,n} e, per i in X, sia X_i = X \\ {i}.
<BR>
<BR>Osservo che, ogni A sott\'insieme proprio di X può essere visto così:
<BR>
<BR>A = int X_i.
<BR>
<BR>[ossia: A si ottiene precisamente cancellando uno alla volta tutti gli elementi che non appartengono ad A].
<BR>
<BR>Questa formula mi permette di stabilire che, fissati i valori di f(X_i), ricavo un unico valore per tutti gli altri sott\'insiemi propri.
<BR>
<BR>Mi resta solo da trovare i vincoli per f(X).
<BR>
<BR>dato che f(X_i) = f( X int X_i ) = min( f(X), f(X_i) ), deduco che deve essere f(X) >= f(X_i) per ogni i.
<BR>
<BR>Quindi la f(-) è univocamente determinata se si specificano f(X) e f(X_i), rispettando il vincolo appena trovato. Del resto, è semplice verificare che, una volta scelti tali valori, la f(-) così costruita è effettivamente una soluzione accettabile.
<BR>
<BR>Contiamo le f(-) trovate: pongo k = f(X); f(X_i) vanno presi liberamente in {1..k}, e quindi ho k^n scelte possibili. Morale, il numero voluto è
<BR>
<BR>sum[k=1..n] (k^n). []
<BR>
<BR>
<BR>\"Maybe. But so great a claim will need to be established, and clear proofs will be required.\"<BR><BR>[ Questo Messaggio è stato Modificato da: marco il 05-01-2005 12:51 ]
[i:2epswnx1]già ambasciatore ufficiale di RM in Londra[/i:2epswnx1]
- - - - -
"Well, master, we're in a fix and no mistake."
Avatar utente
HiTLeuLeR
Messaggi: 1874
Iscritto il: 01 gen 1970, 01:00
Località: Reggio di Calabria

Messaggio da HiTLeuLeR »

Ooops... <IMG SRC="images/forum/icons/icon_razz.gif"> <IMG SRC="images/forum/icons/icon_cool.gif">
Bloccato